Đến nội dung

Hình ảnh

Tìm bài tập đặc sắc về ứng dụng của BDT Bunhia


  • Please log in to reply
Chủ đề này có 16 trả lời

#1
Meveryday

Meveryday

    Hạ sĩ

  • Thành viên
  • 61 Bài viết
Mình đang rất cần các bài tập đặc sắc về BDT Bunhia, các bạn nếu có thì post lên cho mình với. Rất gấp đấy. Thanks trước nha!
Nhat Thi

#2
vdung8CHNams

vdung8CHNams

    Lính mới

  • Thành viên
  • 8 Bài viết
Gia su x,y,z :D 1 va 1/x+1/y+1/z=2.C/m:
$ sqrt{x+y+z} $ :geq $ sqrt{x-1} $+$ sqrt{y-1} $+$ sqrt{z-1} $
terminator

#3
bot

bot

    Lính mới

  • Thành viên
  • 3 Bài viết
dùng cô-si chứng minh bình phương của vế phải bé hơn hoặc bằng
x+y+z-3+2(x+y+z)-6 (1)
thay 2=1/x+1/y+1/z vào pt trên rồi dùng bu-nhi-a ta thấy 2(x+y+z)=(1/x+1/y+1/z)(x+y+z)>=9
(1) >= x+y+z
suy ra ta có bpt tùm lum????????????????????
anh giải bài đi anh!!!!!!!!!!!!!!!!

Bài viết đã được chỉnh sửa nội dung bởi bot: 13-02-2007 - 22:32


#4
Meveryday

Meveryday

    Hạ sĩ

  • Thành viên
  • 61 Bài viết
Bài khác đi bạn ơi, bài nay quen quá rồi, bài đặc sắc hơn đi mấy bạn ơi.
Nhat Thi

#5
bot

bot

    Lính mới

  • Thành viên
  • 3 Bài viết
có đây : cho xyz=1.CMR:x^{2} (y+z)+y^{2} (z+x)+z^{2} (x+y)>=6
hoặc khó hơn là CMR:x(y+z)+y(z+x)+z(x+y)>=6

Bài viết đã được chỉnh sửa nội dung bởi bot: 14-02-2007 - 15:21


#6
hoangmailinh

hoangmailinh

    Binh nhất

  • Thành viên
  • 23 Bài viết
Cho q,b,c dương thỏa ab+bc+ca=1.CMR
:Rightarrow $ \dfrac{a}{bc+1} $ :infty $\dfrac{9}{10}$ :cafe :cafe :clap :clap :infty :)

Bài viết đã được chỉnh sửa nội dung bởi hoangmailinh: 14-02-2007 - 18:10


#7
dtdong91

dtdong91

    Tiến sĩ diễn đàn toán

  • Hiệp sỹ
  • 1791 Bài viết

có đây : cho xyz=1.CMR:x^{2} (y+z)+y^{2} (z+x)+z^{2} (x+y)>=6
hoặc khó hơn là CMR:x(y+z)+y(z+x)+z(x+y)>=6

Nếu đây ko phải là số thực dương thì sai đề kìa x=y=-1,z=1
CÒn nếu x,y,z dương thì quá dễ rùi :Rightarrow
dùng AM_GM có nhay xy+yz+zx :infty 3 :infty
12A1-THPT PHAN BỘI CHÂU-TP VINH-NGHỆ AN

SẼ LUÔN LUÔN Ở BÊN BẠN

#8
dtdong91

dtdong91

    Tiến sĩ diễn đàn toán

  • Hiệp sỹ
  • 1791 Bài viết

Cho q,b,c dương thỏa ab+bc+ca=1.CMR
:Rightarrow $ \dfrac{a}{bc+1} $ :infty $\dfrac{9}{10}$ :cafe :cafe :clap :clap :) :geq

bài này dùng Cauchy-Schwart
$ \sum \dfrac{a^2}{abc+a} \geq \dfrac{(a+b+c)^2}{3abc+a+b+c}$
Bây giờ chỉ đưa về việc c/m abc :infty $ \dfrac{1}{3\sqrt{3}}$ nữa là được
cái này hiển nhiên đúng theo AM_GM
12A1-THPT PHAN BỘI CHÂU-TP VINH-NGHỆ AN

SẼ LUÔN LUÔN Ở BÊN BẠN

#9
tvk

tvk

    Lính mới

  • Thành viên
  • 3 Bài viết
hihi. Bai tap thi vo so ke. Nhung sao ta ko điểm lại xem có bao nhiêu cách chứng minh Bunhi!
IMO 2002 có Bunhi đó.

#10
Meveryday

Meveryday

    Hạ sĩ

  • Thành viên
  • 61 Bài viết
Vậy thì bạn post đề lên đi. Mình cùng thảo luận.
Nhat Thi

#11
Meveryday

Meveryday

    Hạ sĩ

  • Thành viên
  • 61 Bài viết

Nếu đây ko phải là số thực dương thì sai đề kìa x=y=-1,z=1
CÒn nếu x,y,z dương thì quá dễ rùi :perp
dùng AM_GM có nhay xy+yz+zx :D 3 :D

Bài này thuần dùng AM-GM quá bạn ah.
Nhat Thi

#12
hoangmailinh

hoangmailinh

    Binh nhất

  • Thành viên
  • 23 Bài viết
Thêm 1 bài này nữa
Cho a,b,c dương thỏa mãn $ a+b+c=3 $
CMR
$ \sum \dfrac{a^2}{ \sqrt{b+c} } \geq \dfrac{3}{ \sqrt{2 } } $

Bài viết đã được chỉnh sửa nội dung bởi hoangmailinh: 15-02-2007 - 22:52


#13
hoangmailinh

hoangmailinh

    Binh nhất

  • Thành viên
  • 23 Bài viết
1 bài nữa này
Cho a,b,c dương .CNR
$ \sum \dfrac{1}{a(a+2b)} \geq \dfrac{1}{ \sqrt[3]{a^2b^2c^2} } $
Bài này của anh Cuong ben mathnfriend đó .Cách giải đăn giản thui

#14
mysterious

mysterious

    Binh nhất

  • Thành viên
  • 48 Bài viết

có đây : cho xyz=1.CMR:$x^{2} (y+z)+y^{2} (z+x)+z^{2} (x+y) \geq 6$
hoặc khó hơn là CMR:$x(y+z)+y(z+x)+z(x+y)\geq6$

Mình sửa đề o biet' có đúng không

#15
Meveryday

Meveryday

    Hạ sĩ

  • Thành viên
  • 61 Bài viết

Thêm 1 bài này nữa
Cho a,b,c dương thỏa mãn $ a+b+c=3 $
CMR
$ \sum \dfrac{a^2}{ \sqrt{b+c} } \geq \dfrac{3}{ \sqrt{2 } } $

Bài này thì lại vẫn đơn giãn quá bạn ơi, nhìn vào là biết chỉ cần Cauchy_Swatch, rồi Bunhia la xong mà.
Mình cần các bài có cách giải mới lạ, đặc sắc thôi, ví dụ dạng bài mà nhiều Bdt khác khó có thể sử dụng chẳng hạn, hay các bài khó phát hiện, nhận ra đc rằng nó dùng Bunhia là vô cùng nhanh..... đại loại là như vậy đó!
Các bạn tìm giúp mình với nha!
Nhat Thi

#16
Meveryday

Meveryday

    Hạ sĩ

  • Thành viên
  • 61 Bài viết

1 bài nữa này
Cho a,b,c dương .CNR
$ \sum \dfrac{1}{a(a+2b)} \geq \dfrac{1}{ \sqrt[3]{a^2b^2c^2} } $
Bài này của anh Cuong ben mathnfriend đó .Cách giải đăn giản thui


Bài này hay đó bạn ơi, post lời giải lên đi bạn.
Nhat Thi

#17
Meveryday

Meveryday

    Hạ sĩ

  • Thành viên
  • 61 Bài viết

Thêm 1 bài này nữa
Cho a,b,c dương thỏa mãn $ a+b+c=3 $
CMR
$ \sum \dfrac{a^2}{ \sqrt{b+c} } \geq \dfrac{3}{ \sqrt{2 } } $


Các bạn thử giúp mình xem tổng quát bậc của bài này có được không? Không biết có đúng không nhỉ?
Nhat Thi




1 người đang xem chủ đề

0 thành viên, 1 khách, 0 thành viên ẩn danh